Erdős problem 409 ("How many iterations of$n↦\phi(n)+1$ are needed before a prime is reached? Can infinitely many$n$ reach the same prime? What is the density of$n$ which reach any fixed prime?") has been extended to$\mathbb{N}^2$ inthis post, by the nonlinear recurrence$$x_n=1+\frac{\phi(x_{n-1})+\phi(x_{n-2})}2,\;\;\;x_0,x_1\gt2$$where$\phi$ is Euler's totient function.
This extension:
- $(1)$ preserves primes as fixed points of the function;
- $(2)$ ensures that the image of the function is contained in$\mathbb{N}$;
- $(3)$ ensures that the function is indeed a contraction.
I ask if it is possible to do the same in$\mathbb{N}^3$.
A quite natural choice would be$$x_n=1+\frac{\phi(x_{n-1})+2\phi(x_{n-2})+3\phi(x_{n-3})}6$$but, unfortunately, it does not satisfy the second constraint everywhere.
Many thanks.
- $\begingroup$Not sure what you are after. Of course, you can always add a floor or ceiling function to force the result to be an integer.$\endgroup$lulu– lulu2025-11-10 18:08:35 +00:00CommentedNov 10 at 18:08
You mustlog in to answer this question.
Explore related questions
See similar questions with these tags.